Sudoku des intégrales convergentes

13

Réponses

  • Une erreur c'est peut être glisser dans cette évaluation alors :

    $\displaystyle \frac{\int_{0}^{\pi} {\sqrt{\sin(x)} }}{\ 2\sqrt{2}} dx$=$\displaystyle \frac{\Gamma(\frac{3}{4})^2}{(\pi)^{1/2}}$

    https://www.wolframalpha.com/input/?i=definite+integral&rawformassumption={"F",+"DefiniteIntegralCalculator",+"integrand"}+->"sqrt(sin(x))/sqrt(4)"&rawformassumption={"F",+"DefiniteIntegralCalculator",+"variable"}+->"x"&rawformassumption={"F",+"DefiniteIntegralCalculator",+"rangestart"}+->"0"&rawformassumption={"F",+"DefiniteIntegralCalculator",+"rangeend"}+->"pi"&rawformassumption={"C",+"definite+integral"}+->+{"Calculator"}&rawformassumption={"MC",""}->{"Formula"}

    or $\displaystyle \frac{\Gamma(\frac{3}{4})^2}{(\pi)^{1/2}}$=$\displaystyle \frac{\Gamma(\frac{3}{4})}{\Gamma(\frac{1}{4})}\sqrt{\pi*2}$

    la suite en découle
  • Mon message était relatif à la première version de la formule que tu as indiquée.
    Depuis, je n'ai pas vérifié.
  • $J=\displaystyle \int_0^\pi\sqrt{\sin x}dx=\sqrt{2}\int_0^\pi \left(\sin\left(\dfrac{x}{2}\right)\right)^{\tfrac{1}{2}}\left(\cos\left(\dfrac{x}{2}\right)\right)^{\tfrac{1}{2}}dx$

    Appliquer le changement de variable $y=\dfrac{x}{2}$,

    $J=\displaystyle 2\sqrt{2}\int_0^{\tfrac{\pi}{2}}\left(\sin x\right)^{\tfrac{1}{2}}\left(\cos x\right)^{\tfrac{1}{2}}dx$

    $J=\sqrt{2}\beta\left(\dfrac{3}{4},\dfrac{3}{4}\right)=\sqrt{2}\dfrac{\Gamma\left(\dfrac{3}{4}\right)\Gamma\left(\dfrac{3}{4}\right)}{\Gamma\left(\dfrac{3}{2}\right)}$

    et,

    $\Gamma\left(\dfrac{3}{2}\right)=\dfrac{\sqrt{\pi}}{2}$

    donc,

    $\boxed{J=2\sqrt{\dfrac{2}{\pi}}\Gamma\left(\dfrac{3}{4}\right)^2}$
  • Oui désolé j'ai fait quelques manipulations entre temps .
  • Calcul de http://www.les-mathematiques.net/phorum/read.php?4,1236063,1297891#msg-1297891

    On définit pour tout $x\in[0,1]$, la fonction $R$ par,

    $\displaystyle R(x)=\int_0^x \dfrac{\ln t}{1-t}dt=\int_0^1\dfrac{x\ln(xt)}{1-xt}dt$
    Remarquer que, $R$ est continue sur $[0,1]$ et que $R(1)=-\dfrac{\pi^2}{6}$.

    $\displaystyle J=\int_0^1\dfrac{\ln x\ln(1+x^2)}{1-x}dx=\left[R(x)\ln(1+x^2)\right]_0^1-\int_0^1 \dfrac{2xR(x)}{1+x^2}dx$

    $\displaystyle J=-\dfrac{\pi^2\ln 2}{6}-\int_0^1\int_0^1 \dfrac{2x^2\ln(xt)}{(1+x^2)(1-xt)}dtdx$

    $\displaystyle J=-\dfrac{\pi^2\ln 2}{6}-\int_0^1\int_0^1 \dfrac{2x^2\ln x}{(1+x^2)(1-xt)}dtdx-\int_0^1\int_0^1 \dfrac{2x^2\ln t}{(1+x^2)(1-xt)}dtdx$

    $\displaystyle J=-\dfrac{\pi^2\ln 2}{6}+\int_0^1 \left[\dfrac{2x\ln x\ln(1-tx)}{1+x^2}\right]_{t=0}^{t=1} dx-\int_0^1 \left[-\dfrac{t\ln t\ln(1+x^2)}{1+t^2}+\dfrac{2t\ln t\ln(1-tx)}{1+t^2}-\dfrac{2\ln t\ln(1-tx)}{t}\\-\dfrac{2\ln t\arctan x}{1+t^2}\right]_{x=0}^{x=1} dt$


    $\displaystyle J=-\dfrac{\pi^2\ln 2}{6}+\ln 2\int_0^1 \dfrac{t\ln t}{1+t^2}dt+2\int_0^1\dfrac{\ln t\ln(1-t)}{t}dt+\dfrac{\pi}{2}\int_0^1\dfrac{\ln t}{1+t^2}dt$

    $J=-\dfrac{\pi^2\ln 2}{6}-\dfrac{\pi^2\ln 2}{48}+2\zeta(3)-\dfrac{1}{2}\pi G$

    $\boxed{J=-\dfrac{3}{16}\pi^2\ln 2+2\zeta(3)-\dfrac{1}{2}\pi G}$
  • Calcul de l'intégrale http://www.les-mathematiques.net/phorum/read.php?4,1236063,1299383#msg-1299383

    $J=\displaystyle \int_0^1 \dfrac{\ln x\ln(1+x^2)}{1+x}dx=\left[R(x)\ln(1+x^2)\right]_0^1-\int_0^1 \dfrac{R(x)}{1+x}dx$

    $\displaystyle R(x)=\int_0^x \dfrac{\ln t}{1+t}dt=\int_0^1\dfrac{x\ln(tx)}{1+tx}dt$

    Remarquer que $R$ est continue sur $[0;1]$ et que $R(1)=-\dfrac{\pi^2}{12}$


    $\displaystyle J=-\dfrac{1}{12}\pi^2\ln 2-2\int_0^1\int_0^1 \dfrac{x^2\ln(tx)}{(1+tx)(1+x^2)}dtdx$

    $\displaystyle J=-\dfrac{1}{12}\pi^2\ln 2-2\int_0^1\int_0^1 \dfrac{x^2\ln x}{(1+tx)(1+x^2)}dtdx-2\int_0^1\int_0^1 \dfrac{x^2\ln t}{(1+tx)(1+x^2)}dtdx$

    $\displaystyle J=-\dfrac{1}{12}\pi^2\ln 2-2\int_0^1 \left[\dfrac{x\ln x\ln(1+tx)}{1+x^2}\right]_{t=0}^{t=1}dx-\int_0^1 \left[\\
    \dfrac{t\ln t\ln(1+x^2)}{1+t^2}-\dfrac{2t\ln t\ln(1+tx)}{1+t^2}-\dfrac{2\ln t\arctan x}{1+t^2}+\dfrac{2\ln t\ln(1+tx)}{t}\right]_{x=0}^{x=1}dx
    $

    $\displaystyle J=-\dfrac{1}{12}\pi^2\ln 2-\ln 2\int_0^1 \dfrac{t\ln t}{1+t^2}dt+\dfrac{\pi}{2}\int_0^1 \dfrac{\ln t}{1+t^2}dt-2\int_0^1 \dfrac{\ln t\ln (1+t)}{t}dt$

    $J=-\dfrac{1}{12}\pi^2\ln 2+\dfrac{1}{48}\pi^2\ln 2-\dfrac{1}{2}G\pi+\dfrac{3}{2}\zeta(3)$

    $\boxed{J=-\dfrac{1}{16}\pi^2\ln 2-\dfrac{1}{2}G\pi+\dfrac{3}{2}\zeta(3)}$
  • $J=\displaystyle \sum_{k=1}^{\infty} \frac{\zeta(2k)}{k16^{k}}=\ln\left(\Gamma\left(\tfrac{3}{4}\right)\Gamma\left(\tfrac{1}{4}\right)\right)-2\ln 2=\ln \pi -\dfrac{3}{2}\ln 2$

    et,
    $\displaystyle J=4\int_0^{+\infty} \frac{\left(\sinh\left(\frac{x}{8}\right)\right)^2}{x(e^x-1)}dx$
  • Calculs des intégrales http://www.les-mathematiques.net/phorum/read.php?4,1236063,1299721#msg-1299721


    $J=\displaystyle \int_0^1 \dfrac{\ln x\ln(1+x)}{1-x}dx=\Big[R(x)\ln (1+x)\Big]_0^1-\int_0^1 \dfrac{R(x)}{1+x}dx$

    Définir la fonction $R$ pour tout $x\in[0;1]$,

    $\displaystyle R(x)=\int_0^x \dfrac{\ln t}{1-t}dt=\int_0^1 \dfrac{x\ln(xt)}{1-tx}dt$

    Observer que $R$ est continue sur $[0;1]$ and $R(1)=-\dfrac{\pi^2}{6}$.

    $J=\displaystyle -\dfrac{1}{6}\pi^2\ln 2-\int_0^1 \int_0^1\dfrac{x\ln(xt)}{(1-tx)(1+x)}dtdx$

    $J=\displaystyle -\dfrac{1}{6}\pi^2\ln 2-\int_0^1 \int_0^1\dfrac{x\ln(x)}{(1-tx)(1+x)}dtdx-\int_0^1 \int_0^1\dfrac{x\ln(t)}{(1-tx)
    (1+x)}dtdx$

    $J=\displaystyle -\dfrac{1}{6}\pi^2\ln 2+\int_0^1 \left[\dfrac{\ln x\ln(1-tx)}{1+x}\right]_{t=0}^{t=1}dx-\int_0^1 \left[\dfrac{\ln t\ln(1-tx)-\ln t\ln(1+x)}{1+t}-\dfrac{\ln t\ln(1-tx)}{t}\right]_{x=0}^{x=1}dt$

    $J=\displaystyle -\dfrac{1}{6}\pi^2\ln 2+\ln 2\int_0^1 \dfrac{\ln t}{1+t}dt+\int_0^1 \dfrac{\ln t\ln(1-t)}{t}dt$

    $J=\displaystyle -\dfrac{1}{6}\pi^2\ln 2-\dfrac{1}{12}\pi^2\ln 2+\zeta(3)$

    $\boxed{J=\displaystyle -\dfrac{1}{4}\pi^2\ln 2+\zeta(3)}$

    $\displaystyle K=\int_0^1 \dfrac{\ln x\ln(1+x)}{1+x}dx=\Big[S(x)\ln (1+x)\Big]_0^1-\int_0^1 \dfrac{S(x)}{1+x}dx$


    Définir la fonction $S$ pour tout $x\in[0;1]$,

    $\displaystyle S(x)=\int_0^x \dfrac{\ln t}{1+t}dt=\int_0^1 \dfrac{x\ln(xt)}{1+tx}dt$

    Observer que $S$ est continue sur $[0;1]$ et $S(1)=-\dfrac{\pi^2}{12}$.

    $\displaystyle K=-\dfrac{1}{12}\pi^2\ln 2-\int_0^1\int_0^1 \dfrac{x\ln(xt)}{(1+tx)(1+x)}dtdx$

    $\displaystyle K=-\dfrac{1}{12}\pi^2\ln 2-\int_0^1\int_0^1 \dfrac{x\ln(x)}{(1+tx)(1+x)}dtdx-\int_0^1\int_0^1 \dfrac{x\ln(t)}{(1+tx)(1+x)}dtdx$


    $\displaystyle K=-\dfrac{1}{12}\pi^2\ln 2-\int_0^1 \left[\dfrac{\ln x\ln(1+tx)}{1+x}\right]_{t=0}^{t=1}dx-\int_0^1 \left[\dfrac{\ln t\ln(1+x)-\ln t\ln(1+tx)}{t-1}+\dfrac{\ln t\ln(1+tx)}{t}\right]_{x=0}^{x=1}dt$

    $\displaystyle K=-\dfrac{1}{12}\pi^2\ln 2-K+\ln 2\int_0^1 \dfrac{\ln t}{1-t}dt-J-\int_0^1 \dfrac{\ln t\ln(1+t)}{t}dt$

    $\displaystyle 2K=-\dfrac{1}{12}\pi^2\ln 2-\dfrac{1}{6}\pi^2\ln 2+\dfrac{1}{4}\pi^2\ln 2-\zeta(3)+\dfrac{3}{4}\zeta(3)$


    $\boxed{K=\displaystyle -\dfrac{1}{8}\zeta(3)}$


    PS:
    Des carrés avaient disparu, j'ai corrigé les oublis.
  • Un autre intermède,

    $\displaystyle \int_0^1 \dfrac{x\ln x\arctan x}{1+x}dx=\dfrac{1}{2}\ln 2-\dfrac{1}{2}G\ln 2+\dfrac{\pi^3}{64}+\dfrac{\pi^2}{48}-\dfrac{\pi}{4}$
  • Bonjour,

    Ce dernier intermède ne présente pas de difficulté.
    On justifie l'existence de toutes les intégrales.

    Soit $\displaystyle I = \int_{0}^{1} dx {x \ln x \arctan x \over 1+x}.$

    On définit $\displaystyle R(x) = \int_{0}^{x} dt {t \ln t \over 1+t}$ de sorte que $\displaystyle R'(x) = {x \ln x \over 1+x}.$

    On calcule explicitement $\displaystyle R(x) = -Li_2(-x) - x + x \ln x - \ln x \ln(1+x)$ à une constante près.

    On intègre par parties : $\displaystyle I = R(x)\arctan x \mid_{0}^{1} - \int_{0}^{1} dx {R(x) \over 1+x^2}.$

    On ne peut pas intégrer facilement $\displaystyle \int_{0}^{1} dx {R(x) \over 1+x^2}$ sans passer par les complexes alors on va astucer un peu.

    On calcule $\displaystyle R(1) = -Li_2(-1)-1 = {\pi^2 \over 12}-1.$

    On effectue un changement de variables $\displaystyle u = \frac{t}{x}$ pour trouver : $\displaystyle R(x) = \int_{0}^{1} du {x^2 u \ln(ux) \over 1+x u}.$

    On a donc à calculer : $\displaystyle \int_{0}^{1} du {u \over 1+x u} = {1\over x} - {\ln(1+x) \over x^2}.$

    Puis on reporte et on calcule $\displaystyle \int_{0}^{1} dx {x^2 \ln x \over 1+x^2} ({1\over x} - {\ln(1+x) \over x^2}) = \int_{0}^{1} dx {x \ln x \over 1+x^2} - \int_{0}^{1} dx { \ln x \ln(1+x)\over 1+x^2}.$

    La première intégrale se calcule par changement de variable $\displaystyle y=x^2$ et, par exemple, en développant le dénominateur en série ; on trouve facilement : $\displaystyle \int_{0}^{1} dx {x \ln x \over 1+x^2}= - \frac14 {\pi^2 \over 12}.$

    La seconde intégrale est plus coton (avec l'habitude, on le sait) : donc en bon physicien, on la laisse de côté pour le moment.

    Pour calculer l'intégrale restante, on va bien sûr inverser les intégrales sur $x$ et $u$ et on a donc :
    $\displaystyle \int_{0}^{1} du u \ln u \int_{0}^{1} dx {x^2 \over (1+x^2)(1+x u)}$

    L'intégrale sur $x$ se calcule classiquement et on trouve : $\displaystyle \int dx {x^2 \over (1+x^2)(1+x u)} = {u^2 \ln(1+x^2) + 2 \ln(1+x u) - 2u \arctan x \over 2u(1+u^2)}$ à une constante près. Et on on a donc : $\displaystyle \int_{0}^{1} dx {x^2 \over (1+x^2)(1+x u)} = {2 \ln 2 u^2 - \pi u + 4 \ln(1+u) \over 4u(1+u^2)}.$

    On rassemble le tout et on a établi :
    $\displaystyle I = {\pi \over 4} R(1) + {\pi^2 \over 48} + \int_{0}^{1} dx { \ln x \ln(1+x)\over 1+x^2}- {\ln 2 \over 2} \int_{0}^{1} du {u^2 \ln u \over 1+u^2} + {\pi \over 4} \int_{0}^{1} du {u \ln u \over 1+u^2} - \int_{0}^{1} du { \ln u \ln(1+u)\over 1+u^2}.$

    C'est la simplification que l'on attendait ! L'intégrale coton s'élimine.

    On a facilement : $\displaystyle \int_{0}^{1} du {u^2 \ln u \over 1+u^2} = G-1$ et on a déjà trouvé $\displaystyle \int_{0}^{1} du {u \ln u \over 1+u^2} = - {\pi^2 \over 48} .$

    On a donc démontré le résultat que tu indiques.
  • Bravo ! YvesM, merci de l'intérêt que tu portes à ces questions.

    On peut calculer $R(1)$ par développement en série.

    On peut calculer directement $J=\displaystyle \int_0^1 \dfrac{R(x)}{1+x^2}dx=\int_0^1\int_{0}^{1} \dfrac{x^2 u \ln(ux)} {(1+x u)(1+x^2)}dudx$

    $\displaystyle J=\int_{0}^{1} \int_{0}^{1}\dfrac{x^2 u \ln x} {(1+x u)(1+x^2)}du+\int_{0}^{1}\int_{0}^{1} \dfrac{x^2 u \ln u} {(1+x u)(1+x^2)}dudx$


    $\displaystyle J=\int_{0}^{1}\left[-\dfrac{\ln x\ln(1+ux)}{1+x^2}+\dfrac{ux\ln x}{1+x^2}\right]_{u=0}^{u=1}dx+\int_{0}^{1}\left[-\dfrac{\ln u\ln(1+x^2)}{2(1+u^2)}+\dfrac{\ln u\ln(1+x^2)}{2}+\dfrac{\ln u\ln (1+ux)}{1+u^2}\\-\dfrac{u\ln u\arctan x}{1+u^2}\right]_{x=0}^{x=1}du$

    $\displaystyle J=\int_{0}^{1}\dfrac{x\ln x}{1+x^2}dx-\dfrac{1}{2}\ln 2 \int_0^1 \dfrac{\ln u}{1+u^2}du+\dfrac{1}{2}\ln 2\int_0^1 \ln u du-\dfrac{\pi}{4}\int_0^1 \dfrac{u\ln u}{1+u^2}du$

    $\boxed{\displaystyle J=-\dfrac{\pi^2}{48}+\dfrac{1}{2}G\ln 2-\dfrac{1}{2}\ln 2+\dfrac{\pi^3}{192}}$


    PS:
    Ce qui est crucial dans le calcul ci-dessus est que deux intégrales se détruisent mutuellement par somme.
    C'est ce qui fait l'intérêt de cet intégrale et la rend assez aisée à calculer.

    PS2:
    En fait le calcul de YvesM est identique à celui-ci mais je n'aime pas faire appel à la fonction dilogarithme quand je peux m'en passer.
  • En essayant de calculer une intégrale (celle déjà mentionnée plus haut) je suis tombé sur une curiosité (de mon point de vue).

    Une démonstration, presque élémentaire, de l'égalité $\displaystyle \sum_{n=0}^{\infty} \dfrac{(-1)^n}{(2n+1)^3}=\dfrac{\pi^3}{32}$ s'appuyant sur la connaissance de l'égalité $\displaystyle \sum_{n=1}^{\infty} \dfrac{1}{n^2}=\dfrac{\pi^2}{6}$
  • Bonjour,

    Voici une méthode élémentaire et plus rapide pour obtenir $\displaystyle \beta(3) = \sum_{n \geq 0} {(-1)^n \over (2n+1)^3}.$
    Pour tout $n$ entier, on calcule $\displaystyle \int_{0}^{1} dx x^{2n} = {1 \over 2n+1}.$
    On a donc $\displaystyle \beta(3) = \sum_{n \geq 0} \iiint_{[0,1]^3} dxdydz (-x^2y^2z^2)^n.$
    On peut inverser série et intégrale par convergence dominée et on reconnaît une série géométrique :
    $\displaystyle \beta(3) = \iiint_{[0,1]^3} dxdydz {1 \over 1+x^2y^2z^2}.$
    On effectue alors un changement de variable $\displaystyle (x,y,z) = ({\sin u \over \cos v}, {\sin v \over \cos w}, {\sin w \over \cos u})$ dont le jacobien vaut $\displaystyle 1+x^2y^2z^2$ et pour lequel le domaine d'intégration devient $\displaystyle D = \{(u,v,w)| 0 \leq u \leq \frac{\pi}{2} - v, 0 \leq v \leq \frac{\pi}{2} - w, 0 \leq w \leq \frac{\pi}{2} - u\}.$
    On a donc $\displaystyle \beta(3) = \iiint_{D} dudvdw.$ Cette intégrale est $\displaystyle \frac14$ du volume du cube de côté $\displaystyle \frac{\pi}{2}$ : faire un dessin.
    On a donc établi que $\displaystyle \beta(3) =\frac14 (\frac{\pi}{2})^3 = {\pi^3 \over 32}.$
  • Bonjour, YvesM. Le volume du tétraèdre est $1/6$ du volume du cube, n'est-ce pas ?
  • Bonjour,

    Ma géométrie dans l'espace n'est pas ce qu'elle pourrait être. En dessinant le domaine, j'ai commis une erreur. J'ai corrigé mon poste ci-dessus. Le domaine est difficile à décrire (pour moi), alors j'écris ces équations. En faisant un dessin, c'est vraiment $\frac14$ du volume du cube.
  • On peut calculer $\displaystyle \sum_{n=0}^{\infty} \dfrac{(-1)^n}{(2n+1)^3}$ de façon élémentaire (niveau bac+1) en utilisant $\displaystyle \sum_{n=0}^{\infty} \dfrac{(-1)^n}{2n+1}=\dfrac{\pi}4$ (facile à montrer).
    On introduit $I_n=\displaystyle\int_0^{\pi/2}x^2\dfrac{\sin(2nx)}{\sin(x)}dx$.
    $I_{n+1}-I_n=\displaystyle\int_0^{\pi/2}2x^2\cos((2n+1)x)dx=\dfrac{\pi^2(-1)^n}{2(2n+1)}-\dfrac{4(-1)^n}{(2n+1)^3}$ en intégrant par parties.
    $f(x)=\dfrac{x^2}{\sin(x)}$ étant de classe $C^1$ sur $[0,\pi/2]$ on peut intégrer par parties:
    $I_n=\left[-f(x)\dfrac{\cos(2nx)}{2n}\right]_0^{\pi/2}+\dfrac1{2n}\displaystyle\int_0^{\pi/2}f'(x)\cos(2nx)dx$ donc $|I_n|\leq \dfrac{f(\pi/2)+\int_0^{\pi/2}|f'(x)|dx}{2n}$ qui a pour limite 0.
    On en déduit $\displaystyle \sum_{n=0}^{\infty} \dfrac{(-1)^n}{(2n+1)^3}=\dfrac{\pi^2}8\times\dfrac{\pi}4=\dfrac{\pi^3}{32}$.

    En considérant $J_n=\displaystyle\int_0^{\pi/2}x^4\dfrac{\sin(2nx)}{\sin(x)}dx$ on en déduit de manière analogue $\displaystyle \sum_{n=0}^{\infty} \dfrac{(-1)^n}{(2n+1)^5}=\dfrac{5\pi^5}{1536}$.
  • Merci Jandri pour cette méthode. B-)-
  • Dans mon voyage immobile entrepris il y a plus d'un mois pour calculer, de façon élémentaire une intégrale compliquée (mentionnée plus haut), je bute sur l'intégrale suivante, dont je connais une valeur probable par un calcul empirique:

    $\displaystyle \int_0^{\tfrac{\pi}{4}} \ln(\tan x)\ln(\cos x-\sin x)dx=\dfrac{G\ln 2}{2}$

    Où $G$ est la constante de Catalan.

    J'ai l'impression (j'ai eu beaucoup d'impressions fausses depuis le début de ce voyage) que si je trouve une méthode élémentaire pour calculer cette intégrale je termine le calcul (monstrueux) entrepris.

    Merci d'avance pour votre participation.
  • Bonjour,

    $\displaystyle I = \int_{0}^{{\pi \over 4}} dx \ln(\tan x) \ln(\cos x - \sin x).$

    On pose $\displaystyle F(x) = \int_{\frac{\pi}{4}}^{x} dt \ln(\tan t) .$

    On intègre par partie $I$ et on inverse l'ordre d'intégration, et donc $\displaystyle I = \int_{0}^{{\pi \over 4}} dx {\cos x + \sin x \over \cos x - \sin x }F(x) = \int_{0}^{{\pi \over 4}} dt \ln(\tan t) \int_{0}^{t} dx {1+\tan x \over 1-\tan x} .$

    On effectue le changement de variable $\displaystyle u= \tan x$ et $\displaystyle x = \tan t$ pour obtenir $\displaystyle I = \int_{0}^{1} {dx \over 1+x^2} \ln x \int_{x}^{1} {du \over 1+u^2} {1-u \over 1+u}.$ Faux : c'est ${1+u \over 1-u}$

    Je te laisse le plaisir de finir... B-)-
  • Merci de l'intérêt que tu portes à cette question.

    Il y a un truc qui me chagrine.

    On aurait $\displaystyle \int_0^{\tfrac{\pi}{4}}\ln(\tan x)\ln(\cos x-\sin x)dx=\Big[F(x)\ln(\cos x-\sin x)\Big]_0^{\tfrac{\pi}{4}}+\int_0^\tfrac{\pi}{4} F(x)\dfrac{\cos x+\sin x}{\cos x-\sin x}dx$

    Mais:

    $F(0)\ln(\cos 0-\sin 0)=0$

    Mais

    $F(\tfrac{\pi}{4})\ln(\cos \tfrac{\pi}{4}-\sin \tfrac{\pi}{4})$ n'est pas calculable. (l'expression tend vers $+\infty$ quand $x$ se rapproche de $\tfrac{\pi}{4}$ sauf erreur).
  • Prendre $\displaystyle F(x) = \int_{0}^{x} \ln(\tan t)dt+G$ semble être mieux pour appliquer l'intégration par parties.
  • Bonjour,

    Tu as raison. En fait c'est la borne dans $F$ qui est $\frac{\pi}{4}$, mais de toute façon j'ai fait une erreur de signe $1+u$ au numérateur et non pas $1-u$ donc mon calcul est faux.

    Je vais trouver et écrire une démonstration complète, mais c'est toujours très long à écrire...
  • Si je savais calculer, par un moyen simple,

    $\displaystyle \int_0^1\int_0^1 \frac{\mathrm{ln}\left( t\right) x \left( 1+x\right) }{\left( x-1\right) \left( {{x}^{2}}+1\right) \left( {{t}^{2}} {{x}^{2}}+1\right) }dtdx$

    ce serait plié en deux temps, trois mouvements.

    Mais on on ne peut pas intégrer en $x$ d'abord.

    PS:
    J'ai fait une erreur similaire à celle d'YvesM dans une intégration par parties. L'intégrale double ci-dessus ne converge sans doute pas.
  • L'erreur d'YvesM m'a curieusement ouvert des perspectives. Je l'en remercie.

    J'avais cru remarqué que la méthode que j'ai utilisée plusieurs fois dans ce fil de messages comportait la bizarrerie suivante:

    Elle fonctionne bien pour le calcul de $\displaystyle \int_0^1\dfrac{\ln(1+x)\ln x}{1+x}dx$ mais pas pour celui de

    $\displaystyle \int_0^1\dfrac{\ln(1-x)\ln x}{1-x}dx$.

    En fait, si on est assez soigneux, elle fonctionne aussi. :-)


    Soit $R$ la fonction définie sur $[0;1]$ par $\displaystyle R(x)=\int_0^1 \dfrac{\ln x}{1-x}dx=\int_0^1 \dfrac{x\ln(tx)}{1-tx}dt$

    Soit $0<\epsilon<1$,

    $\displaystyle I(\epsilon)=\Big[\left(R(x)-R(1)\right)\ln(1-x)\Big]_0^{1-\epsilon}+\int_0^{1-\epsilon} \dfrac{R(x)-R(1)}{1-x}dx$

    $\displaystyle I(\epsilon)=\left(R(1-\epsilon)-R(1)\right)\ln(\epsilon)+\Big[R(1)\ln(1-x)\Big]_0^{1-\epsilon}+\int_0^{1-\epsilon} \dfrac{R(x)}{1-x}dx$

    $\displaystyle I(\epsilon)=\left(R(1-\epsilon)-R(1)\right)\ln(\epsilon)+R(1)\ln(\epsilon)+\int_0^{1-\epsilon}\left(\int_0^1 \dfrac{x\ln x}{(1-tx)(1-x)}dt\right)dx+\int_0^{1-\epsilon}\left(\int_0^1 \dfrac{x\ln t}{(1-tx)(1-x)}dt\right)dx$


    $\displaystyle I(\epsilon)=\left(R(1-\epsilon)-R(1)\right)\ln(\epsilon)+R(1)\ln(\epsilon)+\int_{0}^{1-\epsilon} \left[-\dfrac{\ln x\ln(1-tx)}{1-x}\right]_{t=0}^{t=1} dx+\int_0^1\left[\dfrac{\ln t\ln(1-tx)}{t}+\\\dfrac{\ln t\ln(1-tx)}{1-t}-\dfrac{\ln t\ln(1-x)}{1-t}\right]_{x=0}^{x=1-\epsilon}dt$

    $\displaystyle I(\epsilon)=\left(R(1-\epsilon)-R(1)\right)\ln(\epsilon)+R(1)\ln(\epsilon)-\int_{0}^{1-\epsilon}\dfrac{\ln x\ln(1-x)}{1-x}dx+\int_0^1 \dfrac{\ln t\ln\left(1-t\left((1-\epsilon\right)\right)}{t} dt+\\\displaystyle\int_0^1 \dfrac{\ln t\ln\left(1-t\left((1-\epsilon\right)\right)}{1-t}dt-R(1)\ln(\epsilon)$

    $\displaystyle I(\epsilon)=\left(R(1-\epsilon)-R(1)\right)\ln(\epsilon)-\int_{0}^{1-\epsilon}\dfrac{\ln x\ln(1-x)}{1-x}dx+\int_0^1 \dfrac{\ln t\ln\left(1-t\left((1-\epsilon\right)\right)}{t} dt+\\\displaystyle\int_0^1 \dfrac{\ln t\ln\left(1-t\left((1-\epsilon\right)\right)}{1-t}dt$

    $\displaystyle \lim_{\epsilon \rightarrow 0} I(\epsilon)=\int_0^1 \dfrac{\ln t\ln\left(1-t\right)}{t} dt=\zeta(3)$


    (En développant en série $\dfrac{\ln(1-x)}{x}$ on peut montrer que cette dernière intégrale est $\displaystyle \sum_{n=1}^{+\infty} \dfrac{1}{n^3}$ )

    Ainsi,

    $\boxed{\displaystyle \int_0^1\dfrac{\ln(1-x)\ln x}{1-x}dx=\zeta(3)}$



    PS:
    Je pensais que $\displaystyle \int_0^1 \dfrac{\ln(1-x)\ln x}{1+x^2}dx$ m'était inaccessible aussi.
    Dans la démarche entreprise j'ai besoin de générer des formules qui lient des quantités et je pense que lorsque j'aurais assez de formules cela deviendra un "simple" calcul algébrique pour terminer. B-)-
    Le paysage se dévoile progressivement au cours de ce voyage...


    PS2:
    Pourquoi j'avais l'impression que cela ne fonctionnait pas?
    Je ne m'occupais pas du crochet dans la formule d'intégration par parties, et je ne me rendais pas compte qu'il ne pouvait pas être évalué, je pensais qu'il valait $0$, je me retrouvais donc avec une intégrale divergente et je n'allais pas chercher plus loin.
    (je fais tous les calculs avec Maxima qui est bien utile pour ce genre de calculs)



    PS3:

    On peut donc en déduire que:

    $\boxed{\displaystyle \int_0^1 \dfrac{\ln(1-x^2)\ln x}{1-x^2}dx=\dfrac{7}{4}\zeta(3)-\dfrac{1}{4}\pi^2\ln 2}$
  • Bonjour,

    Pour démontrer que $\displaystyle I = \int_{0}^{1}dx {\ln(1-x) \ln x \over 1-x} = \zeta(3)$ on peut aussi utiliser une combinaison classique : transformer le logarithme en fraction rationnelle et développer en série entière. On part de $\displaystyle \ln u = \ln x \mid_{1}^{u}$ et on change la variable pour ramener les bornes d'intégration à $0$ et $1$ avec $\displaystyle y = {x-1 \over u-1}$ et alors $\displaystyle \ln u = \int_{0}^{1} dy{-(1-u) \over 1-(1-u) y}$ ; on a donc montré le résultat : $\displaystyle {\ln u \over 1-u} = -\int_{0}^{1} {dt \over 1-(1-u)t}.$ On reporte : $\displaystyle I = \int_{0}^{1} dx {\ln(1-x) \ln x \over 1-x} = -\int_{0}^{1} dx \ln(1-x) \int_{0}^{1} {dt \over 1-(1-x)t}.$ Et là on déroule : changement de variable où $1-x$ devient $x$ et développement en série $\displaystyle {1 \over 1-xt} = \sum_{k \geq 0} (xt)^k.$ On arrive sans peine au résultat.
  • J'imagine que cela revient à faire le changement de variable $y=1-x$

    Et on a donc $\displaystyle \int_0^1\dfrac{\ln x\ln(1-x)}{x}dx$ et on termine avec un développement en série, sachant que,

    $\displaystyle \int_0^1 x^n\ln xdx=-\dfrac{1}{(1+n)^2}$

    C'est plus simple que la méthode que je proposais ci-dessus.

    PS:
    Je pense avoir une preuve (monstrueuse) de la dernière intégrale que j'ai proposée et je pense que la fin du voyage est proche, c'est la dernière étape, me semble-t-il, avant que je puisse prouver la formule donnée pour l'intégrale qui m'occupe depuis plusieurs semaines. Il me manquait deux formules. :-D
  • On veut montrer que,

    \begin{equation*}
    \displaystyle \int_0^{\pi/4} \ln(\tan x)\ln(\cos x-\sin x)dx=\dfrac{G\ln 2}{2}
    \end{equation*}


    \begin{equation*}
    \displaystyle \beta(3)=\sum_{n=1}^{\infty} \dfrac{(-1)^n}{(2n+1)^3}
    \end{equation*}

    et:
    \begin{equation*}
    \displaystyle\int_0^1 \dfrac{\arctan x\ln x}{x}dx=-\beta(3)
    \end{equation*}

    (on développe en série entière $\dfrac{\arctan x}{x}$)

    On pose,
    \begin{align*}
    \displaystyle A&=\int_0^1 \dfrac{x\arctan x\ln x}{1+x^2}dx\\
    \displaystyle B&=\int_0^1 \dfrac{\ln x \ln(1+x^2)}{1+x^2}dx\\
    \displaystyle C&=\int_0^1 \dfrac{\arctan x\ln x}{1+x}dx
    \end{align*}

    Soit $R$ la fonction définie sur $[0;1]$ telle que pour tout $x$ de $[0;1]$,

    \begin{equation*}
    \displaystyle R(x)=\int_0^x \dfrac{\ln t}{1+t^2}dt=\int_0^1 \dfrac{x\ln(tx)}{1+t^2x^2}dt
    \end{equation*}

    Soit $\epsilon$ tel que $0<\epsilon<1$.
    \begin{align*}
    \varphi(\epsilon)&=\int_0^{1-\epsilon}\dfrac{\ln x\ln(1-x)}{1+x^2}dx\\
    &=\Big[\left(R(x)-R(1)\right)\ln(1-x)\Big]_0^{1-\epsilon}+\int_0^{1-\epsilon}\dfrac{\left(R(x)-R(1)\right)}{1-x}dx\\
    &=\displaystyle \left(R(1-\epsilon)-R(1)\right)\ln(\epsilon)+R(1)\ln(\epsilon)+\int_0^{1-\epsilon}\dfrac{R(x)}{1-x}dx\\
    &=\displaystyle \left(R(1-\epsilon)-R(1)\right)\ln(\epsilon)+R(1)\ln(\epsilon)+\int_0^{1-\epsilon} \left(\dfrac{x\ln(tx)}{(1+t^2x^2)(1-x)}dt\right)dx\\
    &=\displaystyle \left(R(1-\epsilon)-R(1)\right)\ln(\epsilon)+R(1)\ln(\epsilon)+\int_0^{1-\epsilon} \left(\int_0^1 \dfrac{x\ln x}{(1+t^2x^2)(1-x)}dt\right)dx+\int_0^1 \left(\int_0^{1-\epsilon} \dfrac{x\ln t}{(1+t^2x^2)(1-x)}dx\right)dt\\
    &=\displaystyle \left(R(1-\epsilon)-R(1)\right)\ln(\epsilon)+R(1)\ln(\epsilon)+\int_0^{1-\epsilon} \left[\dfrac{\ln x\arctan(tx)}{1-x}\right]_{t=0}^{t=1}dx+\\
    &\int_0^1 \left[\dfrac{\ln t\ln(1+t^2x^2)}{2(1+t^2)}+\dfrac{t\ln t\arctan(tx)}{1+t^2}-\dfrac{\ln t\arctan(tx)}{t}-\dfrac{\ln t\ln(1-x)}{1+t^2}\right]_{x=0}^{x=1-\epsilon} dt\\
    &=\displaystyle \left(R(1-\epsilon)-R(1)\right)\ln(\epsilon)+R(1)\ln(\epsilon)+\int_0^{1-\epsilon} \dfrac{\ln x\arctan(x)}{1-x}dx+\int_0^1 \dfrac{\ln t\ln\left(1+t^2(1-\epsilon)^2\right)}{2(1+t^2)}dt+\\
    &\int_0^{1-\epsilon}\dfrac{t\ln t\arctan\left(t(1-\epsilon)\right)}{1+t^2}dt-\int_0^1\dfrac{\ln t\arctan\left(t(1-\epsilon)\right)}{t}dt-\int_0^1\dfrac{\ln t\ln \epsilon}{1+t^2}dt\\
    &=\displaystyle \left(R(1-\epsilon)-R(1)\right)\ln(\epsilon)+\int_0^{1-\epsilon} \dfrac{\ln x\arctan(x)}{1-x}dx+\int_0^1 \dfrac{\ln t\ln\left(1+t^2(1-\epsilon)^2\right)}{2(1+t^2)}dt+\\
    &\int_0^{1-\epsilon}\dfrac{t\ln t\arctan\left(t(1-\epsilon)\right)}{1+t^2}dt-\int_0^1\dfrac{\ln t\arctan\left(t(1-\epsilon)\right)}{t}dt
    \end{align*}

    \begin{equation*}
    \displaystyle\lim_{\epsilon\rightarrow 0}\varphi(\epsilon)=\int_0^{1} \dfrac{\ln x\arctan(x)}{1-x}dx+\int_0^1 \dfrac{\ln t\ln\left(1+t^2\right)}{2(1+t^2)}dt+\int_0^{1}\dfrac{t\ln t\arctan t}{1+t^2}dt-\int_0^1\dfrac{\ln t\arctan t}{t}dt
    \end{equation*}

    Ainsi,

    \begin{equation}
    (1)\boxed{\displaystyle\int_0^{1}\dfrac{\ln x\ln(1-x)}{1+x^2}dx=A+\dfrac{1}{2}B+\beta(3)+\int_0^1\dfrac{\ln x\arctan x }{1-x}dx }
    \end{equation}

    Dans l'intégrale suivante on applique le changement de variable $y=\dfrac{1-x}{1+x}$,

    \begin{align*}
    \displaystyle\int_0^1\dfrac{\ln x\arctan(x)}{1-x}dx&=\int_0^1 \dfrac{\left(\ln(1+x)-\ln(1-x)\right)\arctan\left(\dfrac{x-1}{x+1}\right)}{x}dx+\int_0^1 \dfrac{\left(\ln(1-x)-\ln(1+x)\right)\arctan\left(\dfrac{x-1}{x+1}\right)}{1+x}dx\\
    &=\displaystyle \int_0^1 \dfrac{\ln(1+x)\arctan x}{x}dx-\int_0^1 \dfrac{\ln(1-x)\arctan x}{x}dx-\dfrac{\pi}{4}\int_0^1\dfrac{\ln\left(\tfrac{1+x}{1-x}\right)}{x}dx+\\
    &\int_0^1\dfrac{\ln\left(\tfrac{1-x}{1+x}\right)\arctan\left(\tfrac{x-1}{x+1}\right)}{1+x}dx
    \end{align*}

    et,
    \begin{align*}
    \displaystyle \int_0^1 \dfrac{\ln(1+x)\arctan x}{x}dx&=\Big[\ln x\ln(1+x)\arctan x\Big]_0^1-\int_0^1\ln x\left(\dfrac{\ln(1+x)}{1+x^2}+\dfrac{\arctan x}{1+x}\right)dx\\
    &=\displaystyle -\int_0^1 \dfrac{\ln x\ln(1+x) }{1+x^2}dx-C
    \end{align*}

    \begin{align*}
    \displaystyle \int_0^1 \dfrac{\ln(1-x)\arctan x}{x}dx&=\Big[\ln x\ln(1-x)\arctan x\Big]_0^1-\int_0^1\ln x\left(\dfrac{\ln(1-x)}{1+x^2}-\dfrac{\arctan x}{1-x}\right)dx\\
    &=\displaystyle \int_0^1 \dfrac{\ln x\arctan x }{1-x}dx-\int_0^1 \dfrac{\ln x\ln(1-x) }{1+x^2}dx
    \end{align*}

    Dans l'intégrale qui suit on applique le changement de variable $y=\dfrac{1-x}{1+x}$,

    \begin{align*}
    \displaystyle \int_0^1\dfrac{\ln\left(\tfrac{1+x}{1-x}\right)}{x}dx&=-2\int_0^1\dfrac{\ln x}{1-x^2}dx\\
    &=\dfrac{\pi^2}{4}
    \end{align*}

    Dans l'intégrale qui suit on applique le changement de variable $y=\dfrac{1-x}{1+x}$,

    \begin{equation*}
    \displaystyle \int_0^1\dfrac{\ln\left(\tfrac{1-x}{1+x}\right)\arctan\left(\tfrac{x-1}{x+1}\right)}{1+x}dx=-C
    \end{equation*}

    Ainsi,

    \begin{equation*}
    (2)\boxed{\displaystyle \int_0^1\dfrac{\ln x\arctan(x)}{1-x}dx=\dfrac{1}{2}\int_0^1 \dfrac{\ln x\ln(1-x)}{1+x^2}dx-\dfrac{1}{2}\int_0^1 \dfrac{\ln x\ln(1+x)}{1+x^2}dx-C-\dfrac{\pi^3}{32}}
    \end{equation*}

    Soit $S$ la fonction définie sur $[0;1]$ telle que pour tout $x$ de $[0;1]$,

    \begin{align*}
    \displaystyle S(x)&=\int_0^x\dfrac{\ln x}{1+x^2}dt\\
    &=\int_0^1\dfrac{x\ln(tx)}{1+t^2x^2}dt
    \end{align*}

    Remarquer que $S(1)=-G$,

    \begin{align*}
    \int_0^1\dfrac{\ln x\ln(1+x)}{1+x^2}dx&=\Big[S(x)\ln(1+x)\Big]_0^1-\int_0^1 \dfrac{S(x)}{1+x}dx\\
    &=-G\ln 2-\int_0^1 \int_0^1 \dfrac{x\ln(tx)}{(1+t^2x^2)(1+x)}dtdx\\
    &=-G\ln 2-\int_0^1 \int_0^1 \dfrac{x\ln x}{(1+t^2x^2)(1+x)}dtdx-\!\!\int_0^1 \int_0^1 \dfrac{x\ln t}{(1+t^2x^2)(1+x)}dtdx\\
    &=-G\ln 2-\!\!\!\int_0^1\left[\dfrac{\ln x\arctan(tx)}{1+x}\right]_{t=0}^{t=1}dx-\\
    &\int_0^1\!\!\left[\dfrac{\ln t\ln(1+x^2t^2)}{2(1+t^2)}\!-\!\!\dfrac{\ln t\ln(1+x)}{1+t^2}\!-\!\!\dfrac{t\ln t\arctan(tx)}{1+t^2}\!+\!\!\dfrac{\ln t\arctan(tx)}{t}\right]_{x=0}^{x=1}\!dt\\
    &=-G\ln 2-\int_0^1\dfrac{\ln x\arctan x}{1+x}dx-\dfrac{1}{2}\int_0^1\dfrac{\ln t\ln(1+t^2)}{1+t^2}dt+\\
    &\ln 2\int_0^1\dfrac{\ln t}{1+t^2}dt+\int_0^1\dfrac{t\ln t\arctan t}{1+t^2}dt-\int_0^1\dfrac{\ln t\arctan t}{t}dt\\
    \end{align*}

    Ainsi,

    \begin{equation*}
    (3)\boxed{\displaystyle \int_0^1\dfrac{\ln x\ln(1+x)}{1+x^2}dx=A-\dfrac{1}{2}B-C-2G\ln 2+\beta(3)}
    \end{equation*}


    Soit $T$ la fonction définie sur $[0;1]$ telle que pour tout $y$ de $[0;1]$,

    \begin{align*}
    \displaystyle T(y)&=\int_0^y \dfrac{t\ln(t)}{1+t^2}dt\\
    &=\displaystyle\int_0^1 \dfrac{ty^2\ln(ty)}{1+t^2y^2}dt
    \end{align*}

    Remarquer que $T(1)=-\dfrac{\pi^2}{48}$

    \begin{align*}
    \displaystyle A&=\Big[T(y)\arctan y\Big]_0^1-\int_0^1 \dfrac{S(y)}{1+y^2}dy\\
    &=\displaystyle-\dfrac{\pi^3}{192}-\int_0^1\int_0^1\dfrac{ty^2\ln(ty)}{(1+t^2y^2)(1+y^2)}dtdy\\
    \displaystyle &=-\dfrac{\pi^3}{192}-\int_0^1\int_0^1\dfrac{ty^2\ln y }{(1+t^2y^2)(1+y^2)}dtdy-\int_0^1\int_0^1\dfrac{ty^2\ln t }{(1+t^2y^2)(1+y^2)}dtdy\\
    \displaystyle &=-\dfrac{\pi^3}{192}-\dfrac{1}{2}\int_0^1\left[\dfrac{\ln y \ln(1+t^2y^2)}{1+y^2}\right]_{t=0}^{t=1}dy-\\
    &\dfrac{1}{2}\int_0^1 \left[\dfrac{\ln t \arctan y+\ln t\arctan(ty)}{1+t}-\dfrac{\ln t \arctan(ty)-\ln t\arctan y}{t-1}\right]_{y=0}^{y=1}dt
    \end{align*}

    Ainsi,
    \begin{align*}
    (4)\boxed{\displaystyle A=-\dfrac{1}{64}\pi^3-\dfrac{1}{2}B-\dfrac{1}{2}C-\dfrac{1}{2}\int_0^1\dfrac{\ln x\arctan x}{1-x}dx}
    \end{align*}


    On remplace l'expression de $\displaystyle \int_0^{1}\dfrac{\ln x\ln(1-x)}{1+x^2}dx$ donnée par (1) et celle de $\displaystyle \int_0^{1}\dfrac{\ln x\ln(1+x)}{1+x^2}dx$ donnée par (4) dans (2),

    Ainsi,
    \begin{align*}
    (5)\boxed{\displaystyle \int_0^1\dfrac{\ln x\arctan x}{1-x}dx=B-C+2G\ln 2-\dfrac{1}{16}\pi^3}
    \end{align*}

    On remplace l'expression de $\displaystyle \int_0^1\dfrac{\ln x\arctan x}{1-x}dx$ donnée par (5) dans (4),

    \begin{align*}
    (6)\boxed{A=\dfrac{1}{64}\pi^3-B-G\ln 2}
    \end{align*}

    En appliquant le changement de variable $y=\arctan x$ dans l'intégrale suivante,

    \begin{align*}
    (7) \boxed{\displaystyle \int_0^{\pi/4} \ln(\tan x)\ln(\cos x-\sin x)dx=\int_0^1\dfrac{\ln x\ln(1-x)}{1+x^2}dx-\dfrac{B}{2}}
    \end{align*}

    et, de (1), on déduit que:

    \begin{align*}
    (8) \boxed{\displaystyle \int_0^{\pi/4} \ln(\tan x)\ln(\cos x-\sin x)dx=A+\beta(3)-\int_0^1\dfrac{\ln x\arctan x}{1-x}dx}
    \end{align*}

    On remplace $A$ donnée par (6) et $\displaystyle \int_0^1\dfrac{\ln \arctan x}{1-x}dx$ donnée par (5) dans (7),

    \begin{align*}
    (8) \boxed{\displaystyle \int_0^{\pi/4} \ln(\tan x)\ln(\cos x-\sin x)dx=-\dfrac{3}{64}\pi^3+G\ln 2+\beta(3)-C}
    \end{align*}

    $C$ a été calculée dans un message plus haut,

    \begin{equation}
    \boxed{\displaystyle C=\dfrac{G\ln 2}{2}-\dfrac{\pi^3}{64}}
    \end{equation}

    et, sachant que,

    \begin{equation}
    \beta(3)=\dfrac{\pi^3}{32}
    \end{equation}

    on obtient,

    \begin{equation}
    \boxed{\displaystyle \int_0^{\pi/4} \ln(\tan x)\ln(\cos x-\sin x)dx=\dfrac{G\ln 2}{2}}
    \end{equation}
  • Ce qui suit est une preuve élémentaire que,


    $$\displaystyle I=\int_0^1 \dfrac{x\ln(1-x^2)\arctan x}{1+x^2}dx=G\ln 2-\dfrac{1}{48}\pi^3-\dfrac{1}{8}\pi\left(\ln 2\right)^2$$

    \begin{align}
    \displaystyle I&=\int_0^1 \dfrac{x\ln(1-x^2)\arctan x}{1+x^2}dx\\
    \displaystyle &=\int_0^1 \dfrac{x\ln(1+x)\arctan x}{1+x^2}dx+\int_0^1 \dfrac{x\ln(1-x)\arctan x}{1+x^2}dx
    \end{align}

    Soit,
    \begin{equation}
    \displaystyle F=\int_0^1 \dfrac{x\ln(1-x)\arctan x}{1+x^2}dx
    \end{equation}


    Appliquer le changement de variable $y=\dfrac{1-x}{1+x}$ dans l'intégrale précédente,

    $\displaystyle F=\int_0^1 \dfrac{\Big(\ln 2-y\ln 2+(1-y)\ln y+(y-1)\ln (1+y)\Big)\arctan\left(\dfrac{1-y}{1+y}\right)}{y^3+y^2+y+1}dy$


    Pour $y\neq -1$, definir la fonction $H$,
    \begin{equation}
    \displaystyle H(y)=\dfrac{\Big(\ln 2-y\ln 2+(1-y)\ln y+(y-1)\ln (1+y)\Big)\arctan\left(\dfrac{1-y}{1+y}\right)}{y^3+y^2+y+1}
    \end{equation}

    Puisque $0<y<1$,

    \begin{equation}
    \arctan\left(\dfrac{1-y}{1+y}\right)=\dfrac{\pi}{4}-\arctan y
    \end{equation}

    et, pour tout $y\neq 1$,

    $\dfrac{1}{y^3+y^2+y+1}=\dfrac{1}{(1+y)(1+y^2)}=\dfrac{1}{2(1+y)}+\dfrac{1-y}{2(1+y^2)}$

    $\dfrac{y}{y^3+y^2+y+1}=\dfrac{y}{(1+y)(1+y^2)}=\dfrac{1+y}{2(1+y^2)}-\dfrac{1}{2(1+y)}$

    alors, pour tout $y\neq -1$,

    \begin{align*}
    H&=\dfrac{\Big(\big(-\ln 2-\ln y+\ln(1+y)\big)y+\big(\ln 2+\ln y-\ln(1+y)\big)\Big)\Big(\dfrac{\pi}{4}-\arctan y\Big)}{y^3+y^2+y+1}\\
    &=-\dfrac{y\arctan y\ln(1+y)}{1+y^2}+\dfrac{\arctan y\ln(1+y)}{1+y}+
    \dfrac{\pi y\ln(1+y)}{4(1+y^2)}-\dfrac{\pi \ln(1+y)}{4(1+y)}+\\
    &\dfrac{y\arctan y\ln y}{1+y^2}-\dfrac{\arctan y\ln y}{1+y}-\dfrac{\pi y\ln y}{4(1+y^2)}+\dfrac{\pi \ln y}{4(1+y)}+\dfrac{y\ln 2\arctan y}{1+y^2}-\\
    &\dfrac{\ln 2\arctan y}{1+y}-\dfrac{\pi y\ln 2}{4(1+y^2)}+\dfrac{\pi \ln 2}{4(1+y)}
    \end{align*}

    Soient,
    \begin{align*}
    \displaystyle A&=\int_0^1 \dfrac{x\arctan x\ln x}{1+x^2}dx\\
    \displaystyle B&=\int_0^1 \dfrac{\ln x \ln(1+x^2)}{1+x^2}dx\\
    \displaystyle C&=\int_0^1 \dfrac{\arctan x\ln x}{1+x}dx\\
    \displaystyle J&=\int_0^1\dfrac{\arctan x\ln(1+x)}{1+x}dx
    \end{align*}

    Ainsi,

    \begin{equation}
    (1)\boxed{\displaystyle I=A-C+J-\dfrac{5}{384}\pi^3-\dfrac{7}{32}\pi\left(\ln 2\right)^2+\dfrac{1}{2}G\ln 2}
    \end{equation}

    $G$, étant la constante de Catalan.

    $$\int_0^1 \dfrac{\arctan x\ln(1-x)}{1+x}dx-\int_0^1\dfrac{\arctan x\ln(1+x)}{1+x}dx=\int_0^1 \dfrac{\arctan x\ln\left(\dfrac{1-x}{1+x}\right)}{1+x}dx$$


    Appliquer le changement de variable $y=\dfrac{1-x}{1+x}$,

    $$\int_0^1 \dfrac{\arctan x\ln(1-x)}{1+x}dx-\int_0^1\dfrac{\arctan x\ln(1+x)}{1+x}dx=\int_0^1 \dfrac{\ln x\arctan\left(\dfrac{1-x}{1+x}\right)}{1+x}dx$$

    Ainsi,

    \begin{align}
    \int_0^1 \dfrac{\arctan x\ln(1-x)}{1+x}dx-\int_0^1\dfrac{\arctan x\ln(1+x)}{1+x}dx&=\dfrac{\pi}{4}\int_0^1\dfrac{\ln x}{1+x}dx-\int_0^1\dfrac{\ln x\arctan x}{1+x}dx\\
    &=-\dfrac{\pi^3}{48}-C
    \end{align}

    Ainsi,

    $$(2)\boxed{\int_0^1 \dfrac{\arctan x\ln(1-x)}{1+x}dx=J-\dfrac{\pi^3}{48}-C}$$



    Appliquer le changement de variable $y=1-x$,

    $$\int_0^1 \dfrac{\arctan x\ln(1-x)}{1+x}dx=\int_0^1 \dfrac{\arctan(1-x) \ln(x)}{2-x}dx$$


    Définir la fonction $R$ sur $[0;1]$ par,

    $$R(x)=\int_0^x \dfrac{\ln t}{2-t}dt=\int_0^1 \dfrac{x\ln(tx)}{2-tx}dt$$

    \begin{align}
    \int_0^1 \dfrac{\arctan(1-x) \ln(x)}{2-x}dx&=\Big[R(x)\arctan(1-x)\Big]_0^1+\int_0^1\int_0^1\dfrac{x\ln(tx)}{(2-tx)(1+(1-x)^2)}dtdx-\int_0^1\left[\dfrac{\ln x\ln(2-tx)}{1+(1-x)^2}\right]_{t=0}^{t=1}dx+\\
    &\displaystyle\int_0^1\left[\dfrac{\ln t\ln(x^2-2x+2)}{2(1+(1-t)^2)}-\dfrac{\ln t\ln(2-tx)}{1+(1-t)^2}-\dfrac{t\ln t\arctan(x-1)}{1+(1-t)^2}+\dfrac{\ln t\arctan(x-1)}{1+(1-t)^2}\right]_{x=0}^{x=1}dt\\
    &=\ln 2\int_0^1\dfrac{\ln x}{1+(1-x)^2}dx-\int_0^1 \dfrac{\ln x\ln(2-x)}{1+(1-x)^2}dx-\dfrac{\ln 2}{2}\int_0^1 \dfrac{\ln t}{1+(1-t)^2}dt+\\
    &\ln 2\int_0^1\dfrac{\ln t}{1+(1-t)^2}dt-\int_0^1\dfrac{\ln t\ln(2-t)}{1+(1-t)^2}dt-\dfrac{\pi}{4}\int_0^1 \dfrac{t\ln t}{1+(1-t)^2}dt+\dfrac{\pi}{4}\int_0^1 \dfrac{\ln t}{1+(1-t)^2}dt
    \end{align}


    Appliquer le changement de variable $y=1-x$,

    \begin{align}
    \displaystyle \int_0^1 \dfrac{\arctan(x) \ln(1-x)}{1+x}dx&=\ln 2\int_0^1\dfrac{\ln(1-x)}{1+x^2}dx-\int_0^1 \dfrac{\ln (1-x)\ln(1+x)}{1+x^2}dx-\dfrac{\ln 2}{2}\int_0^1 \dfrac{\ln (1-t)}{1+t^2}dt+\\
    &\ln 2\int_0^1\dfrac{\ln(1-t)}{1+t^2}dx-\int_0^1 \dfrac{\ln (1-t)\ln(1+t)}{1+t^2}dt-\dfrac{\pi}{4}\int_0^1\dfrac{(1-t)\ln(1-t)}{1+t^2}dt+\\
    &\dfrac{\pi}{4}\int_0^1 \dfrac{\ln (1-t)}{1+t^2}dt\\
    \displaystyle &=\dfrac{3}{2}\ln 2\int_0^1 \dfrac{\ln (1-t)}{1+t^2}dt+\dfrac{\pi}{4}\int_0^1 \dfrac{t\ln (1-t)}{1+t^2}dt-2\int_0^1 \dfrac{\ln (1-x)\ln(1+x)}{1+x^2}dx\\
    \displaystyle&=\dfrac{3}{2}\ln 2\left(\dfrac{\pi\ln 2}{8}-G\right)+\dfrac{\pi}{4}\left(\dfrac{(\ln 2)^2}{8}-\dfrac{5\pi^2}{96}\right)-2\int_0^1 \dfrac{\ln (1-x)\ln(1+x)}{1+x^2}dx
    \end{align}

    Ainsi,

    \begin{equation*}
    (3)\boxed{\displaystyle \int_0^1 \dfrac{\arctan(x) \ln(1-x)}{1+x}dx=\dfrac{7}{32}\pi(\ln 2)^2-\dfrac{3}{2}G\ln 2-\dfrac{5\pi^3}{384}-2\int_0^1 \dfrac{\ln (1-x)\ln(1+x)}{1+x^2}dx\\}
    \end{equation*}




    Appliquer la formule d'intégration par parties dans l'intégrale suivante,

    \begin{equation*}
    \displaystyle \int_0^1 \dfrac{(\ln(1+x))^2}{1+x^2}dx=\Big[\arctan x\left(\ln\left(1+x\right)\right)^2\Big]_0^1-2\int_0^1 \dfrac{\arctan x\ln(1+x)}{1+x}dx
    \end{equation*}

    \begin{equation*}
    (4)\boxed{\displaystyle \int_0^1 \dfrac{(\ln(1+x))^2}{1+x^2}dx=\dfrac{\pi}{4}(\ln 2)^2-2J}
    \end{equation*}


    Appliquer le changement de variable $y=\dfrac{1-x}{1+x}$ dans l'intégrale suivante,

    \begin{align}
    \displaystyle \int_0^1 \dfrac{\ln(1+x)\ln x}{1+x^2}dx&=\int_0^1\dfrac{(\ln(1+x))^2}{1+x^2}dx-\int_0^1 \dfrac{\ln(1-x)\ln(1+x)}{1+x^2}dx+\ln 2\int_0^1 \dfrac{\ln(1-x)}{1+x^2}dx-\\
    &\ln 2\int_0^1 \dfrac{\ln(1+x)}{1+x^2}dx
    \end{align}

    Ainsi,

    \begin{align*}
    \displaystyle \int_0^1 \dfrac{\ln(1-x)\ln(1+x)}{1+x^2}dx&=\int_0^1\dfrac{(\ln(1+x))^2}{1+x^2}dx-\int_0^1 \dfrac{\ln(1+x)\ln x}{1+x^2}dx+\ln 2\int_0^1 \dfrac{\ln\left(\tfrac{1-x}{1+x}\right)}{1+x^2}dx\\
    &=\int_0^1\dfrac{(\ln(1+x))^2}{1+x^2}dx-\int_0^1 \dfrac{\ln(1+x)\ln x}{1+x^2}dx-G\ln 2\\
    \end{align*}


    Et donc,

    $$(5)\boxed{\displaystyle \int_0^1 \dfrac{\ln(1-x)\ln(1+x)}{1+x^2}dx=\dfrac{\pi}{4}(\ln 2)^2-G\ln 2-\int_0^1 \dfrac{\ln(1+x)\ln x}{1+x^2}dx-2J}$$


    Remplacer (5) dans (3),

    \begin{align*}
    \begin{split}
    \displaystyle \int_0^1 \dfrac{\arctan(x) \ln(1-x)}{1+x}dx=-\dfrac{9}{32}\pi(\ln 2)^2+\dfrac{1}{2}G\ln 2-\dfrac{5\pi^3}{384}+2\int_0^1 \dfrac{\ln x\ln(1+x)}{1+x^2}dx+4J
    \end{split}
    \end{align*}


    En utilisant (2) il vient,


    \begin{equation*}
    (6)\boxed{\displaystyle J=\dfrac{\pi^3}{384}-\dfrac{G\ln 2}{3}+\dfrac{3}{32}\pi(\ln 2)^2-\dfrac{2}{3}\int_0^1\dfrac{\ln x\ln(1+x)}{1+x^2}dx}
    \end{equation*}


    On a vu plus haut que:

    $$\displaystyle \int_0^1\dfrac{\ln x\ln(1+x)}{1+x^2}dx=A-\dfrac{1}{2}B-C-2G\ln 2+\beta(3)$$

    et on sait que, $\displaystyle \beta(3)=\sum_{n=1}^{\infty} \dfrac{(-1)^n}{(2n+1)^3}$

    Il vient,

    $$(7)\boxed{J=\dfrac{\pi^3}{384}+G\ln 2+\dfrac{3\pi\left(\ln 2\right)^2}{32}-\dfrac{2}{3}A+\dfrac{1}{3}B+\dfrac{2}{3}C-\dfrac{2}{3}\beta(3)}$$


    On a vu plus haut que,

    $$(8)\boxed{A=\dfrac{1}{64}\pi^3-B-G\ln 2}$$

    Il vient,

    $$(9)\boxed{J=\dfrac{5}{3}G\ln 2-\dfrac{\pi^3}{128}+\dfrac{3\pi\left(\ln 2\right)^2}{32}+B+\dfrac{2}{3}C-\dfrac{2}{3}\beta(3)}$$


    Remplacer (8) et (9) dans (1) il vient,

    $$(10)\boxed{I=-\dfrac{1}{192}\pi^3+\dfrac{7}{6}G\ln 2-\dfrac{1}{8}\pi\left(\ln 2\right)^2-\dfrac{1}{3}C-\dfrac{2}{3}\beta(3)}$$


    On a vu plus haut que,

    $$C=\dfrac{G\ln 2}{2}-\dfrac{\pi^3}{64}$$

    et sachant que, $\beta(3)=\dfrac{\pi^3}{32}$,

    Il vient,

    $$\boxed{I=G\ln 2-\dfrac{1}{48}\pi^3-\dfrac{1}{8}\pi\left(\ln 2\right)^2}$$

    fin de partie, c'est la fin du voyage...
  • J'essaie de trouver une forme close pour l'intégrale suivante.

    $$\displaystyle \int_0^1 \dfrac{\ln x\ln(1+x)\arctan x}{1+x}dx$$

    Je poste l'énoncé par acquis de conscience car peut-être qu'une méthode simple que je ne vois pas permet de calculer cette intégrale même si j'en doute et permet d'éviter des calculs monstrueux.
    Pour le moment dans ce calcul entrepris je ne rencontre que des "connaissances" sympathiques.

    PS:
    Pourquoi le choix de cette intégrale?
    Parce qu'on sait calculer

    $$\displaystyle \int_0^1 \dfrac{\ln x\arctan x}{1+x}dx$$
    et:
    $$\displaystyle \int_0^1 \dfrac{\ln x\ln(1+x)}{1+x}dx$$

    et j'y vois des signes encourageants que l'intégrale considérée est sympathique malgré son apparence peu engageante. :-)
  • Intermède,

    Si on regarde trop vite l'intégrale,

    $\displaystyle \int_0^1 \dfrac{\left(\ln(1+x)\right)^2}{1+x}dx$

    on a l'impression qu'elle est monstrueuse alors qu'elle est gentille tout plein. J'aime cette intégrale :-)
  • Bonjour dans le cadre de mon cours je dois calculer le plus simplement possible l'intégrale suivante (sans utiliser de récurrence ni de fonctions spéciales):
    $$\int_{0}^{\pi/2}sin(x)^ndx$$
    Du coup j'ai la piste suivante

    $$\int_{0}^{\pi/2}sin(x)^ndx=\int_{0}^{\pi/2}cos(x)^ndx=\int_{0}^{\pi/2}sin(2x)^ndx=C\int_{0}^{\pi/2}sin(kx)^n$$ ou C est une constante et k appartient aux entiers naturels

    Cordialement
  • $\sin x=\dfrac{e^{ix}-e^{-ix}}{2i}$

    PS:
    Si a,b sont non nuls il existe c tel que a=cb.

    Ta constante C elle ne dépend pas de n?
  • Juste une question Fin de partie à quoi est égale l'intégrale suivante :

    $$\int_{0}^{\pi/2}sin(kx)^n$$

    Ps:
    Je proposerai une intégrale plus difficile pour faire fuir la trivialité de mon précédent post .
  • Je n'ai pas compris l'intérêt de cette suite d'égalités:

    $\displaystyle \int_{0}^{\pi/2}sin(x)^ndx=\int_{0}^{\pi/2}cos(x)^ndx=\int_{0}^{\pi/2}sin(2x)^ndx=C\int_{0}^{\pi/2}sin(kx)^n$

    On peut prendre $k=1$ et $C=1$
  • C'est en vue d'utiliser la formule de la moyenne d'une fonction continue :
    Il existe c appartenant à [a,b] tel que :

    $$\frac{1}{b-a}\int_{a}^{b}f(t)dt=f(c)$$

    Dans notre cas $a=0$ et $b=\frac{\pi}{2}$ et $f(t)=sin(t)$ et $g(t)=cos(t)$ on trouve :


    $$\frac{1}{\frac{\pi}{2}-0}\int_{0}^{\frac{\pi}{2}}sin(t)^ndt=f(c)$$ et comme :

    $$\int_{0}^{\pi/2}sin(t)^ndt=\int_{0}^{\pi/2}cos(t)^ndt$$ on obtient :

    $$\frac{1}{\frac{\pi}{2}-0}\int_{0}^{\frac{\pi}{2}}cos(t)^ndt=g(z)=f(c)$$

    on a donc $sin(c)^n=cos(z)^n$ d'ou $c=z=\frac{\pi}{4}$ mais ce résultat me semble faux...Fin de partie une suggestion ?

    Donc $$\frac{1}{\frac{\pi}{2}-0}\int_{0}^{\frac{\pi}{2}}sin(t)^ndt=f(\frac{\pi}{4})$$
  • Je ne connais pas de cas dans une utilisation non frelatée de ce théorème de la moyenne qui permette d'obtenir une valeur explicite (un nombre algébrique bien identifié, un nombre rationnel multiplié par Pi....).

    Les intégrales que tu cherches se calculent par récurrence. Cela me semble la même meilleure façon.
  • Merci de partager ton expérience sur le sujet .Pour rester dans le sujet d'ailleurs je propose l'intégrale suivante:

    $$\int_{-\infty}^{0}\frac{((ln(cos(e^{x})+sin(e^{x}))(-1)e^{cos(x)}))}{e^{sin(x)}}$$

    Cordialement
  • Si c'est une intégrale que tu as créée en écrivant au hasard des symboles il y a très peu de chance qu'elle ait une autre écriture qui ait un intérêt.
    Si tu connais une telle autre écriture merci de l'indiquer.
  • Cette intégrale convergente je l'ai trouvé seul effectivement . La méthode de Laplace dans le cas générale permettrait au moins d'avoir un équivalent de l'intégrale...
    .
  • On ne s'est pas compris.

    L'intégrale d'une fonction continue sur un intervalle où elle est continue donne toujours une valeur finie.
    Mais est-ce que cette valeur est remarquable? Est-ce que cette valeur est égale à la valeur évaluée en des constantes de l'analyse, comme $\pi$, d'un polynôme à coefficients rationnels à plusieurs variables?

    Ce sont ces intégrales avec valeurs remarquables qui m'intéressent. Les autres sont vulgaires et de peu d'intérêt à mon sens. B-)-
  • Un exemple d'intégrale intéressante (pour moi et d'autres semble-t-il) dont je ne sais pas justifier la valeur probable. :-D

    $\displaystyle \int_0^{\pi/2}{\frac{1+2\cos 2x\cdot\ln\tan x}{1+\tan^{2\sqrt{2}} x}}\tan^{1/\sqrt{2}} x~dx=0$
  • Ah au fait tu pourrais m'aider à faire l'exercice suivant ( en échange je te dénicherai une intégrale remarquable) :55726
  • Et l'intégrale comme prévu :

    $$ \int_{\pi/6}^{\pi/2}\bigg[\Re\big(\text{Li}_2(4\sin^2\theta)\big) +\text{Li}_2\bigg(\frac{1}{4\sin^2\theta}\bigg) \bigg]d\theta=\frac{5\pi^3}{54}. $$

    Cordialement (:-D)
  • Dans ton exercice on te dit quoi faire, au moins pour la première question.

    la fonction exponentielle admet un développement en série entière, $\displaystyle \sum_{n=0}^{+\infty} \dfrac{z^n}{n!}$ pour toute valeur du plan complexe qui converge (au moins) ponctuellement vers cette fonction.

    Commence par écrire ce que cela donne.
  • Quelle est une période de la fonction f?

    Jamais entendu parlé de:
    https://fr.wikipedia.org/wiki/Égalité_de_Parseval#Formule_pour_les_s.C3.A9ries_de_Fourier

    ?

    Et pour la troisième question, tu as un développement en série entière dans le membre de droite en fonction de $a$ qu'est-ce qu'on a envie de faire dans le membre de gauche?

    PS:
    La seule difficulté, toute relative, est de justifier l'interversion d'une intégrale et du signe somme d'une série.
  • On a :

    $$e^{ae^{ix}}=\sum_{n=0}^{+\infty}\frac{(ae^{ix})^n}{n!}$$

    L'égalité de départ est (on pose ensuite le changement de variable $x+\pi=t$:

    $$\frac{1}{2\pi}\int_{0}^{2\pi}|f(t)|^2dt=\sum_{n=-\infty}^{\infty}|c_n(f)|^2$$

    L'égalité de parseval transformée est alors:

    $$\frac{1}{2\pi}\int_{-\pi}^{\pi}|f(x+\pi)|^2dx=\sum_{n=0}^{\infty}|c_n(f)|^2$$

    Les coefficient de Fourier sont les $\frac{a^k}{k!}$ dans le cas qui nous concerne :

    On a alors :
    $$\frac{1}{2\pi}\int_{-\pi}^{\pi}|e^{acos(x+\pi)}|^2dx=\sum_{n=0}^{\infty}|\frac{a^k}{k!}|^2$$

    ce qui achève la question 2)

    Pour la dernière question je sèche quelques pistes néanmoins :

    $$cos(x)=\sum_{k=0}^{+\infty}(-1)^k\frac{x^{2k}}{(2k)!}$$
    et
    $$cos(x)^2=\sum_{k=0}^{+\infty}\frac{x^{4k}}{((2k)!)^2}$$
    on pose $k=\frac{n}{2}$ ce qui implique :
    $$cos(x)^2=\sum_{n=0}^{+\infty}\frac{x^{2n}}{((n)!)^2}$$
  • Ce que tu as fait dans la première question tu peux le refaire dans la dernière question. ;-)

    Tu ne vois pas d'où on va sortir les $I_{2n}$ du membre de gauche de l'égalité dans la question 2) ?
  • Vraiment je suis un cake Fin de partie je ne vois vraiment pas ou aller ....Mais j'ai trouvé ceci d'intéressant55730
  • Une remarque pour la question 2).

    Sauf erreur, quand tu intègres une fonction T périodique qui est intégrable, la valeur obtenue ne change pas si tu intègres sur n'importe quel intervalle de longueur T. Tu peux donc considérer l'intervalle $[-\pi,\pi]$ au lieu de l'intervalle $[0,2\pi]$.

    Le membre de gauche de l'égalité dans la question 2) est une fonction en a.
    Tu peux développer en série entière la fonction en a qui est sous l'intégrale (comme fait pour la question 1) puis intégrer.
    Tu auras besoin de justifier le fait que tu peux intervertir le signe somme de l'intégrale et celui de la série entière.

    Tu auras donc une égalité de deux développements en série entière. Que peut-on dire des coefficients de ces séries?
  • Mon professeur m'a tout expliqué brièvement . Il y a effectivement un interversion somme-intégrale à faire et vérifier s'il y a convergence uniforme(il y a une histoire de compacité derrière cela) . Ensuite comme tu la dis il y a une correspondance entre les différents facteurs qui apparaissent. Merci pour les explications Fin de partie . (tu)
Connectez-vous ou Inscrivez-vous pour répondre.